Mở rộng cho trường hợp đa thức với hệ số trên miền UFD

Một phần của tài liệu (LUẬN văn THẠC sĩ) tiêu chuẩn eisenstein về tính bất khả quy của đa thức (Trang 31 - 48)

UFD

Sau đây là mở rộng Tiêu chuẩn Eisenstein cho trường hợp đa thức với hệ số trên miền UFD (xem [6]).

V là một miền nguyên. Ta kí hiệu FracV là trường các thương của V, nghĩa là FracV !a b |a, b PV, b 0 ) là trường bé nhất chứa V. Ví dụ 2.3.1 Frac Z Q.

Định lý 2.3.2 (Tiêu chuẩn Eisenstein). Cho V là một UFD, F FracV, và fpxq PVrxs với fpxq a0 a1x ... anxn. Nếu có một phần tử nguyên tố p P V sao cho

i) p - an,

ii) p | ai với 0 ¤ i  n, và iii) p2 - a0

thì fpxq bất khả quy trên F.

Trước khi chứng minh Định lý, ta chú ý rằng Bổ đề Gauss có thể mở rộng cho đa thức trên miền phân tích duy nhất. Cụ thể, nếufpxq PVrxs có phân tích fpxq gpxqhpxq với gpxq, hpxq P FracV, thì tồn tại hai đa thức

g1pxq, h1pxq P Vrxs sao cho fpxq g1pxqh1pxq và deggpxq degg1pxq,

deghpxq degh1pxq.

Đặt ppq tpa | aP Vu là iđêan chính sinh bởi p. Do p nguyên tố nên

ppq là iđêan tối đại.

Chứng minh. Đặt V V{ppq. Vì ppq là một iđêan tối đại trong V nên V

là một trường. Với r P V, cho r là ảnh của r mod p; và với g P Vrxs, cho

g ÞÑ g là một đồng cấu từ Vrxs đến Vrxs. Giả sử ngược lại là f phân tích thành gh trong Frxs. Khi đó, theo Bổ đề Gauss, ta có thể lấy g, hP Vrxs, trong đó hai đa thức g và h đều có bậc dương. Vì ta đang xét Vrxs, ta có ánh xạ phân tích f gh vào Vrxs : f gh. Nhưng f anxn 0, vì p là ước của mọi hệ số của f trừ an. Như vậy f phân tích thành tích các đa thức bất khả quy: anxx x. Vì Vrxs là một UFD nên phân tích này là duy nhất, và do đó g cxl, và h dxm.

Hai số l và m là gì? Ta so sánh chúng với degg và degh. Rõ ràng là

l ¤ degg và m ¤ degh. Nhưng khi đó

degf l m¤ degg degh degf

và do degf degf (theo trên), ta có degg l và degh m, trong đó

l, m¡ 0. Vì các hệ số tự do của g và h là 0, ta suy ra các hệ số tự do của

g và h chia hết cho p. Nhưng hệ số tự do của f là tích của các hệ số tự do của g và h, do đó nó chia hết cho p2, mâu thuẫn. l

Lưu ý rằng, nếu fpxq P Vrxs là đa thức nguyên bản và bất khả quy trên FracV thì fpxq bất khả quy trên V.

Ví dụ 2.3.3 Các đa thức sau là bất khả quy trên Zris. i) fpxq x4 p2 3iqx3 11 10i.

ii) gpxq p1 iqx3 p4 iqx2 2p5iqx 4p1 iq.

Lời giải.

i) fpxq x4 p2 3iqx3 11 10i có các hệ số a4 1, a3 2 3i,

a2 a1 0, a0 11 10i. Ta có Zris là miền phân tích duy nhất (xem Ví dụ 2.2.3). Xét số nguyên tố p 2 3i trong Zris (vì a2 b2

22 32 13 là số nguyên tố trong Z nên p là nguyên tố trong Zris

theo Bổ đề 2.2.8). Ta có a0 11 10i p2 3iqp4 iq. Khi đó p

là ước của ai, i 0, ..,3, p không là ước của a4 1 (phần tử khả nghịch trong Zris là và chỉ là 1,i theo Bổ đề 2.2.7). Có p2 không là ước của a0 p2 3iqp4 iq (do Zris là miền phân tích duy nhất

và 2 3i,4i là các số nguyên tố nên a0 có phân tích duy nhất như thế). Vậy theo Tiêu chuẩn Eisenstein trên miền phân tích duy nhất Zris với số nguyên tố p 2 3i thì đa thức fpxq bất khả quy trên Zris (Định lý 2.3.2).

ii) gpxq p1 iqx3 p4 iqx2 2p5iqx 4p1 iq. Xét

gpx iq p1 iqx3 p1 4iqx2 p5 3iqx 3 12i

p1 iqx3 p1 4iqx2 p1 4iqp1iqx 3p1 4iq.

Chọn phần tử nguyên tố trong Zris là p 1 4i (theo Bổ đề 2.2.8). Ta thấy p là ước của các hệ số của gpx iq trừ hệ số của số hạng cao nhất và p2 không là ước của số hạng tự do. Do đó theo Định lý 2.3.2 thì đa thức gpx iq bất khả quy trên Zris. Suy ra, theo Bổ đề 1.1.2 thì đa thức gpxq bất khả quy trên Zris.

Ví dụ 2.3.4 Chứng minh đa thức hai biến

fpx, yq x5 x4y2 x4 x2y42x y42y2 2

bất khả quy trên Q.

Lời giải. Ta viết fpx, yq px2 1qy4 px42qy2 px42qpx 1q gpyq

là đa thức biến y trên Qrxs. Ta có Qrxs là miền phân tích duy nhất. Chọn phần tử bất khả quy trong Qrxs là ppxq x4 2 (theo Tiêu chuẩn Eisenstein với p 2, xem Định lý 1.2.1). Ta có ppxq là ước của các hệ số của gpyq trừ hệ số của số hạng cao nhất và p2pxq không là ước của số hạng tự do. Do vậy theo Tiêu chuẩn Eisenstein trên miền phân tích duy nhất thì gpyq bất khả quy trên Qrxs (Định lý 2.3.2). Vậy đa thức fpx, yq bất khả quy trên Q.

2.4 Vận dụng xét tính bất khả quy của đa thức

Trong tiết này chúng ta vận dụng các kiến thức trình bày ở trên để giải quyết một số bài toán về tính bất khả quy của đa thức, trong đó có các đề

thi IMO (International Mathematical Olympiad, kì thi Toán học cấp quốc tế hàng năm dành cho học sinh trung học phổ thông), VMO (Viet Nam Mathematical Olympiad, kì thi chọn học sinh giỏi quốc gia môn Toán lớp 12 THPT), TST (Team Selection Test, kì thi chọn học sinh vào Đội tuyển học sinh dự thi IMO).

2.4.1. Dạng 1: Dùng Tiêu chuẩn Eisenstein Bài tập 1. Xét tính bất khả quy trên Q của đa thức a) fpxq 2x5 12x46x3 18x2 30x24.

b) gpxq 10x4 14x2 21x 56. c) hpxq x2018 2019.

Lời giải.

a) fpxq 2x5 12x46x3 18x2 30x24.

Chọn số nguyên tố p 3. Ta thấy p 3 là ước của tất cả các hệ số của

fpxq trừ hệ số của số hạng cao nhất, p2 9 không là ước của số hạng tự do. Theo Tiêu chuẩn Eisenstein thì fpxq bất khả quy trên Q.

b) gpxq 10x4 14x221x 56. Chọn số nguyên tố p 7. Ta thấy p 7

là ước của tất cả các hệ số của gpxq trừ hệ số của số hạng cao nhất,

p2 49 không là ước của số hạng tự do. Theo Tiêu chuẩn Eisenstein thì gpxq bất khả quy trên Q.

c) hpxq x2018 2019. Ta có 2019 3673.

Chọn p 3, theo Tiêu chuẩn Eisenstein thì hpxq bất khả quy trên Q.

Nhận xét: Tổng quát cho các đa thức dạng này, chúng ta có thể xem

Hệ quả 1.2.5.

Bài tập 2. Xét tính bất khả quy trên Q của đa thức a) x4 2x3 4x2 6x 5.

b) x6 11x5 51x4 127x3 179x2135x 43.

Lời giải.

a) fpxq x4 2x3 4x2 6x 5.

Ta có fpx 1q x4 6x3 16x2 24x 18.

Chọn số nguyên tố p 2. Theo tiêu chuẩn Eisenstein thì fpx 1q bất khả quy trên Q. Do đó fpxq bất khả quy trên Q (theo Bổ đề 1.1.2). b) gpxq x6 11x5 51x4 127x3 179x2 135x 43.

Ta có gpx 2q x6 x5 x4 x3 x2 x 1, đây là đa thức chia đường tròn thứ 7. Theo Hệ quả 1.2.4 thì gpx 2q bất khả quy trên Q. Vậy gpxq bất khả quy trên Q (theo Bổ đề 1.1.2).

Bài tập 3. (Việt Nam TST 2013). Tìm tất cả các số nguyên dương

n ¡ 1 và số nguyên tố p sao cho đa thức Ppxq xnpx p2 khả quy trên tập các số nguyên Z.

Lời giải. Giả sử Ppxq anxn ... a1x a0 khả quy trên Z, tức là viết được Ppxq gpxqhpxq, trong đó gpxq brxr ... b1x b0 P Zrxs;

hpxq csxs ... c1x c0 P Zrxs với r, s  n. Đồng nhất hệ số của đẳng thức Ppxq gpxqhpxq ta được

ai b0ci b1ci1 bic0 với mọi i 0,1, , n.

Từ b0c0 p2, ta xét hai trường hợp:

• Nếu chỉ có b0 hoặc c0 chia hết cho p. Không mất tính chất tổng quát ta giả sử p | b0 và p - c0. Vì p - an 1 nên tồn tại chỉ số m sao cho

p -bm. Gọi k là chỉ số bé nhất mà p - bk. Ta có

ak b0ck b1ck1 bk1c1 bkc0.

Suy ra ak không chia hết cho p (do bk và c0 không chia hết cho p). Do đó k n. Lại vì k ¤ r   n, ta suy ra vô lí.

• Nếu b0 và c0 cùng chia hết cho p, ta có thể giả sử b0 c0 p. Ta xét tiếp hai trường hợp nhỏ:

– Nếu các đa thứcgpxqvàhpxqđều có bậc không nhỏ hơn2, thì ta có

p b0c1 b1c0 vàb0c2 b1c1 c0b2 0, suy rab1 c1 1vàb1c1 chia hết cho p. Do đó ta có thể giả sửb1 chia hết chop vàc1 không chia hết cho p (vì b1 c1 1). Gọi bk là hệ số đầu tiên của gpxq

không chia hết chop. Từ hệ thứcak 1 bk 1c0 bkc1 ... b0ck 1, với chú ý c0 chia hết cho p, c1 không chia hết cho p và theo cách chọn bk ta suy ra ak 1 không chia hết cho p, điều này vô lí (do

k 1   n nên ak 1 0).

– Do vậy đểPpxqkhả quy thì phải có ít nhất một đa thức gpxqhoặc

hpxq có bậc bằng 1. Nói cách khác, Ppxq phải có nghiệm nguyên là x0. Khi đó x0 chỉ có thể nhận giá trị là 1;p;1;p;p2;p2. Thay vào Ppxq ta suy ra p phải chẵn, do đó p 2. Kiểm tra lại ta chỉ có n 3 thỏa mãn.

Bài tập 4. (VMO 1997). Hãy tìm một đa thức ppxq với hệ số hữu tỷ và bậc càng nhỏ càng tốt thỏa mãnp ?3

3 ?3

9

3 ?3

3. Hỏi rằng có tồn tại hay không đa thức qpxqvới hệ số nguyên thỏa mãn q ?3

3 ?39 9 3 ?3 3? Lời giải. Đặt α ?3 3 ?3 9. Ta thấy pα 1qp?3 3 1q 3 1. Suy ra 3 ?

3pα 1q α 3. Lập phương hai vế của đẳng thức trên ta được

3pα 1q3 pα 3q3 suy ra α3 9α12 0.

Do đó α là nghiệm của đa thức kpxq x3 9x 12. Áp dụng Tiêu chuẩn Eisenstein với số nguyên tố p 3 thì kpxq là đa thức bất khả quy trên Q. Suy ra kpxq x3 9x 12 là đa thức có bậc nhỏ nhất nhận ?3

3 ?3

9 làm nghiệm. Giả sử tồn tại đa thức ppxq P Qrxs thỏa mãn p ?3

3 ?3

9

3 ?3

3. Lấy đa thức ppxq chia cho kpxq ta được

ppxq kpxqgpxq rpxq, trong đó gpxq, rpxq P Qrxs và rpxq 0 hoặc degrpxq   3. Suy ra ppαq kpαqgpαq rpαq. Vì p ?3 3 ?3 9 3 ?3 3

nên ta có r ?3

3 ?3

9

3 ?3

3. Do đó trường hợp rpxq 0 không xảy ra. Với degrpxq  3, ta có hai trường hợp sau:

Trường hợp 1: degrpxq 1. Đặt rpxq ax b với a, b P Q. Khi đó ta có

ap?3 3 ?3 9q b 3 ?3 3. Suy ra pa1q?3 3 a?3 9 3b. Do 3b P Q nên a1 0 và a 0, điều này là vô lí.

Trường hợp 2: degrpxq 2. Đặt rpxq ax2 bx c với a, b, c P Q. Khi đó ta có ap?3 3 ?3 9q2 bp?3 3 ?3 9q c 3 ?3 3. Suy ra pa bq?3 9 p3a bq?3 3 6a c 3 ?3 3. Do đó $ ' ' & ' ' % a b 0 3a b 1 6a c 3 . Vì thế $ ' ' ' ' & ' ' ' ' % a 1 2 b 1 2 c 0 . Ta được rpxq 1 2x 2 1 2x. Khi đó ppxq kpxqgpxq 1 2x 2 1 2x. Vậy ta chọn đa thức ppxq 1 2x 2 1 2x có hệ số hữu tỷ và có bậc bé nhất thỏa mãn p ?3 3 ?3 9 3 ?3 3. Giả sử tồn tại đa thức qpxq P Zrxs thỏa mãn q ?3

3 ?3

9

3 ?3

3. Vì đa thức kpxq PZrxs và có hệ số cao nhất bằng 1 nên lấy qpxq chia cho kpxq

ta được qpxq kpxqhpxq tpxq trong đóhpxq, tpxq PZrxs và tpxq 0 hoặc

degtpxq   3. Lập luận tương tự như trên dẫn đều đến vô lí. Vậy không tồn tại đa thức hệ số nguyên qpxq mà f ?3

3 ?3

9

3 ?3

3.

Bài tập 5. (VMO 2017). Tồn tại hay không đa thức ppxq với hệ số nguyên thỏa mãn pp1 ?3

2q 1 ?3

2 và pp1 ?

5q 2 3?

5?

Lời giải. Giả sử tồn tại đa thức ppxq thỏa mãn yêu cầu bài toán. Đặt

α 1 ?3

2. Khi đó α 1 ?3

2. Lập phương hai vế của đẳng thức đó ta được α3 3α3 3α 3 0. Do đó α là nghiệm của đa thức

qpxq x3 3x2 3x3. Áp dụng Tiêu chuẩn Eisenstein với số nguyên tố p 3 thì qpxq là đa thức bất khả quy. Suy ra qpxq x3 3x2 3x3

là đa thức có bậc nhỏ nhất nhận α là nghiệm. Vì ppxq, qpxq P Zrxs và

qpxq có hệ số cao nhất bằng 1 nên lấy ppxq chia cho qpxq ta được ppxq qpxqfpxq rpxq trong đó fpxq, rpxq PZ và rpxq 0 hoặc degrpxq  3. Vì

pp1 ?3 2q 1 ?3 2 nên rp1 ?3 2q 1 ?3 2. Đặt rpxq ax2 bx c với a, b, c P Z. Khi đó ta có ap1 ?3 2q2 bp1 ?3 2q c 1 ?3 2. Suy ra a?3 4 p2a bq?3 2 a b c 1 ?3 2. Vì vậy $ ' ' & ' ' % a 0 2a b 1 a b c 1. Từ đó ta được $ ' ' & ' ' % a 0 b 1 c 0. Suy ra rpxq x. Do đó ta có ppxq qpxqfpxq x. (2.1) Tương tự như trên, ta đặt β 1 ?

5, suy ra β là nghiệm của đa thức bất khả quy kpxq x22x4 P Zrxs. Lấy đa thức ppxq chia cho kpxq ta được ppxq kpxqgpxq spxq trong đó gpxq, spxq P Zrxs và spxq 0 hoặc

degspxq   2. Vì fp1 ? 5q 2 3? 5 nên sp1 ? 5q 2 3? 5. Từ đó tìm được spxq 3x1. Do đó, ta có ppxq kpxqgpxq 3x1. (2.2) Từ (2.1) và (2.2) suy ra qpxqfpxqkpxqgpxq 2x1. Đến đây, chọnx 8

sẽ suy ra điều mâu thuẫn vì vế trái chia hết cho 11, còn vế phải thì không. Vậy không tồn tại đa thức qpxq thỏa mãn yêu cầu bài toán.

2.4.2. Dạng 2: Dùng tiêu chuẩn Eisenstein mở rộng

Bài tập 6. Chứng minh đa thức fpxq 5x20 5x9 20x 2 bất khả quy trên Q.

Lời giải. fpxq 5x20 5x9 20x 2 có số hạng tự do a0 2, hệ số của số hạng cao nhất a20 5. Ta áp dụng Tiêu chuẩn Eisenstein mở rộng trên Z (Hệ quả 2.1.7) cho đa thứcfpxqvới số nguyên tốp 5:p - a0, p2 -a20, p |ai

với mọi i 1,2, ...,20. Suy ra đa thức fpxq bất khả quy trên Q.

Bài tập 7. Chứng minh đa thức gpxq 5x36x2 p84iqx 1 i bất khả quy trên Zris.

Lời giải. Ta có Zris là miền phân tích duy nhất và có

gcdp5,6,84i,1 iq 1.

Chọn phần tử nguyên tố p 1 i P Zris. Ta thấy p là ước của tất cả các hệ số của gpxq trừ hệ số của số hạng cao nhất, p2 2i không là ước của số hạng tự do. Theo tiêu chuẩn Eisenstein mở rộng trên miền phân tích duy nhất thì gpxq bất khả quy trên Zris.

Bài tập 8. Chứng minh đa thức fpxq xn 5x 25 bất khả quy trên Q với mọi n ¥2.

Lời giải. Áp dụng Định lý 2.1.1, đa thức fpxq xn 5x 25 có an

1, a1 5, a0 25. Ta chọn số số nguyên tố p 5, khi đó p 5 không

Một phần của tài liệu (LUẬN văn THẠC sĩ) tiêu chuẩn eisenstein về tính bất khả quy của đa thức (Trang 31 - 48)

Tải bản đầy đủ (PDF)

(48 trang)